AUD study

अब Quizwiz के साथ अपने होमवर्क और परीक्षाओं को एस करें!

Which of the following procedures would an auditor be least likely to use in an effort to obtain evidence regarding subsequent events? Investigating personnel changes which occurred after year-end. Reading minutes of board meetings and reading interim financial statements. Making inquiry of management about unusual adjustments after year-end. Obtaining lists of litigation for review with the client's attorneys.

Choice "1" is correct. Personnel changes generally would not have financial statement implications. Choice "2" is incorrect. The auditor should read the minutes of board meetings and examine the latest available interim financial statements to determine whether there are any items that might have financial statement implications. Choice "3" is incorrect. The auditor should inquire of management regarding any material unusual adjustments made after year-end, because such adjustments might require financial statement adjustment or disclosure. Choice "4" is incorrect. The auditor should inquire of the client's legal counsel concerning litigation, claims, and assessments, because such issues might have financial statement implications.

Henderson, CPA has decided to use probability-proportional-to-size (PPS) sampling, sometimes referred to as dollar-unit or cumulative-dollar-amount sampling, in the audit of a client's accounts receivable. Henderson will use the following PPS sampling table: Reliability Factors for Errors of Overstatement Number of Overstatement Misstatements Risk of Incorrect Acceptance 1% 5% 10% 0 4.61 3.00 2.31 1 6.64 4.75 3.89 2 8.41 6.30 5.33 3 10.05 7.76 6.69 4 11.61 9.16 8.00 Additional Information Tolerable misstatement (net of effect of expected misstatement) 126,000 Risk of incorrect acceptance 5% Number of misstatements allowed 2 Recorded amount of accounts receivable 700,000 Number of accounts 500 What sample size should Henderson use? 126 70 140 35

Explanation $126,000 $700,000Choice "4" is correct. The reliability factor, as determined from the table, is 6.3. Sampling interval is then calculated as tolerable misstatement divided by the reliability factor, or $126,000 / 6.3 = $20,000. In a population of $700,000 with a sampling interval of $20,000, the sample size will be $700,0000 / $20,000, or 35 accounts. Choices "3", "1", and "2" are incorrect, based on the above explanation.

Which of the following types of audit evidence is the least reliable? Purchase order. Bank statement obtained from the client. Canceled check. Vendor's invoice

Explanation Choice "1" is correct. A purchase order is internal documentation and as such, it is more easily manipulated by the client. Choices "4", "3", or "2" are incorrect. A vendor's invoice, a canceled check, and a bank statement obtained from the client are all considered external evidence, which is less likely to be manipulated than is internal evidence.

Which of the following items would most likely require an adjustment to the financial statements for the year ended December 31, Year 1? Loss on an uncollectible trade receivable recorded in Year 1 from a customer that declared bankruptcy in Year 2. Uninsured loss of inventories purchased in Year 1 as a result of a flood in Year 2. Settlement of litigation in Year 2 over an event that occurred in Year 2. Proceeds from a capital stock issuance in Year 2 which was being approved by the board of directors in Year 1.

Explanation Choice "1" is correct. A trade receivable existing at the date of the financial statements that subsequently becomes uncollectible because of bankruptcy of the customer requires an adjustment to the financial statements. Choice "2" is incorrect. Significant business events occurring subsequent to the date of financial statements, such as a loss resulting from a flood, require no adjustment to the statements, but may require significant additional disclosure. Choice "3" is incorrect. Settlement in Year 2 of litigation related to a Year 2 event would not require adjustment to the Year 1 financial statements, but would require disclosure in the financial statements. Choice "4" is incorrect. Proceeds from a capital stock issuance in Year 2 would not require adjustment to the Year 1 financial statements, but would require disclosure in the financial statements.

Which of the following statements is true about required procedures in a review of annual financial statements? Assessment of fraud risk is not required, but obtaining a representation letter from management is required. Obtaining an understanding of internal control is not required, but confirmations of accounts receivable is required. Obtaining an understanding of the client's operations is not required, but corroboration of management's material estimates is required. Analytical review procedures are not required, but communication with the predecessor accountant is required

Explanation Choice "1" is correct. Assessment of fraud risk is not required, but obtaining a representation letter from management is required. Choice "2" is incorrect. While it is true that obtaining an understanding of internal control is not required in a review of annual financial statements, confirmation of accounts receivable is an audit procedure that is not required in a review engagement. Note: Obtaining an understanding of internal control is required in a review of interim financial statements (when the annual financial statements are audited). Choice "3" is incorrect. Obtaining an understanding of the client's operations is required, and corroboration of management's material estimates is an audit procedure that is not required in a review engagement. Choice "4" is incorrect. Analytical review procedures are required, whereas communication with the predecessor accountant is not required.

Which of the following auditor concerns would be least likely to cause the auditor to withdraw from the engagement? A material error was found in the depreciation calculation for the current period. The industry in which the client operates is burdened by excessive competition and client management is dominated by an individual who emphasizes the achievement of specific profit margins. The integrity of management is in question. Upper level of management appears to have engaged in collusion in an effort to embezzle receipts on account.

Explanation Choice "1" is correct. Errors are unintentional and would not cause an auditor to withdraw from the engagement. Choice "2" is incorrect. A high degree of competition coupled with the fact that a single individual dominates management increases the risk of fraud. In cases where significant fraud risk exists, the auditor may consider withdrawing from the engagement. Choices "3" or "4" are incorrect. Concerns about management integrity may cause the auditor to withdraw from the engagement, because such concerns increase the likelihood of financial statement misrepresentation.

The auditor concluded that the disclosures made in the financial statements did not adequately inform financial statement users about the company's ability to continue as a going concern. Under these circumstances, the auditor should issue a(n): Qualified (except for) or adverse opinion, depending on the level of materiality. Adverse opinion or a disclaimer of opinion. Qualified (except for) or disclaimer of opinion depending on the level of materiality. Unmodified opinion with an emphasis-of-matter paragraph or disclaimer of opinion, depending on the level of materiality.

Explanation Choice "1" is correct. Improper disclosure of a going concern situation is a GAAP violation, so a qualified or adverse opinion should be issued. Choices "3" and "2" are incorrect. While the auditor is never precluded from issuing a disclaimer of opinion, the direct result of inadequate disclosure is either a qualified or adverse opinion. Choice "4" is incorrect. The opinion would be unmodified with an emphasis-of-matter paragraph when there is a going concern issue that has been adequately disclosed. The auditor is never precluded from issuing a disclaimer of opinion, but this is not the general result of inadequate disclosure.

Which of the following best describes a difference between a compilation of prospective financial statements and an examination of prospective financial statements? A compilation does not contemplate the auditor providing any form of assurance whereas an examination does include assurance. An examination includes a statement of positive assurance whereas a compilation includes a statement of negative assurance. In an examination engagement, the practitioner should update his or her report for known changes occurring after the date of the report, whereas there is no similar requirement for a compilation engagement. The compilation engagement is covered under SAS while the examination engagement is covered under SSAE.

Explanation Choice "1" is correct. In an examination engagement, an auditor provides positive assurance regarding whether the statements are presented in conformity with AICPA guidelines, and whether the underlying assumptions provide a reasonable basis for the financial statements. In a compilation engagement, no assurance is provided. Choice "2" is incorrect. In a compilation engagement, no assurance is provided. Choice "3" is incorrect. Regardless of whether the engagement is a compilation or an examination, the practitioner has no responsibility to update his/her report for changes occurring after the date of the report. Choice "4" is incorrect. Examinations on prospective financial statements are covered under SSAE. Compilations of prospective financial statements are covered under SSARS (not SAS).

Which of the following procedures would an auditor most likely perform to verify management's assertion of completeness? Compare a sample of shipping documents to related sales invoices. Review standard bank confirmations for indications of kiting. Observe the client's distribution of payroll checks. Confirm a sample of recorded receivables by direct communication with the debtors.

Explanation Choice "1" is correct. In order to verify management's assertion of completeness, an auditor would most likely compare a sample of shipping documents to related sales invoices to determine that all goods shipped were properly included in sales. Choice "2" is incorrect. Reviewing standard bank confirmations does not provide evidence of kiting. Kiting involves interbank transfers, and bank confirmations only show year-end balances, not transfers between banks. Choice "3" is incorrect. Observing the client's distribution of payroll checks would test management's assertion regarding existence of employees. Choice "4" is incorrect. Confirming a sample of recorded receivables by direct communication with the debtors (i.e., sending accounts receivable confirmations) would verify management's assertion of existence of the accounts receivable.

Which of the following most likely would be an internal control procedure designed to detect errors and irregularities concerning the custody of inventory? Independent comparisons of finished goods records with counts of goods on hand. Approval of inventory journal entries by the storekeeper. Segregation of functions between general accounting and cost accounting. Periodic reconciliation of work in process with job cost sheets.

Explanation Choice "1" is correct. Independently comparing inventory records with physical inventory counts may detect discrepancies concerning the custody of inventory. Choice "2" is incorrect. Requiring approval of journal entries relates to the recordkeeping aspects of inventory and not to its physical custody. Choice "3" is incorrect. Segregation of duties between general accounting and cost accounting relates to the recordkeeping aspects of inventory and not to its physical custody. Choice "4" is incorrect. Periodic reconciliation of work in process with job cost sheets relates to the recordkeeping aspects of inventory and not to its physical custody.

Which of the following properly describes the auditor's responsibilities as opposed to management's responsibilities? Management is responsible for affirming that the effects of any uncorrected misstatements in the financial statements are immaterial and the auditor is responsible for obtaining reasonable assurance about whether the financial statements are free of material misstatement. The auditor is responsible for identifying the laws and regulations applicable to the entity's activities and management is responsible for affirming that the effects of any uncorrected misstatements in the financial statements are immaterial. The auditor is responsible for the entity's financial statements and management is responsible for the selection and application of accounting principles. Management is responsible for the entity's financial statements and the auditor is responsible for the selection and application of accounting principles.

Explanation Choice "1" is correct. Management is responsible for affirming that the effects of any uncorrected misstatements in the financial statements are immaterial and the auditor is responsible for obtaining reasonable assurance about whether the financial statements are free of material misstatement. Choice "2" is incorrect. Management is responsible for both identifying the laws and regulations applicable to the entity's activities, and for affirming that the effects of any uncorrected misstatements in the financial statements are immaterial. Choices "3" and "4" are incorrect. Management is responsible for both the entity's financial statements and for the selection and application of accounting principles.

According to the profession's ethical standards, a CPA would be considered independent in which of the following instances? The CPA belongs to a country club client in which membership requires the acquisition of a pro rata share of equity. The CPA owns an office building and the mortgage on the building is guaranteed by a client. A client leases part of an office building from the CPA, resulting in a material indirect financial interest to the CPA. The CPA has a material direct financial interest in a client, but transfers the interest into a blind trust.

Explanation Choice "1" is correct. Membership in a social club in which membership requirements involve acquisition of a pro rata share of equity does not impair independence because such equity is not considered to be a direct financial interest. The member, however, should not serve in any management capacity. Choice "2" is incorrect. A CPA's independence is considered impaired if the CPA has any loan to or from the client. A loan includes a guarantee of a loan. Choice "3" is incorrect. Leasing property to a client results in an indirect financial interest that impairs a CPA's independence. Choice "4" is incorrect. Any direct or material indirect financial interest impairs a CPA's independence with respect to a client, whether or not the financial interest is placed in a blind trust.

Which of the following entity-level controls is specifically identified in the professional standards as a control of importance that should be evaluated? Period-end financial reporting controls Centralized processing controls Monitoring of controls Risk-management policies

Explanation Choice "1" is correct. Period-end financial reporting process controls are specifically identified within the professional standards as items of importance that should be evaluated. Choice "2" is incorrect. Centralized processing controls are entity-level controls, but are not specifically identified within the professional standards as items of importance that should be evaluated. Period-end financial reporting process controls and the control environment are specifically identified within the professional standards as items of importance that should be evaluated. Choice "3" is incorrect. Monitoring of controls is an entity-level control, but is not specifically identified within the professional standards as items of importance that should be evaluated. Period-end financial reporting process controls and the control environment are specifically identified within the professional standards as items of importance that should be evaluated. Choice "4" is incorrect. Risk management policies are entity-level controls, but are not specifically identified within the professional standards as items of importance that should be evaluated. Period-end financial reporting process controls and the control environment are specifically identified within the professional standards as items of importance that should be evaluated.

An auditor uses variables sampling techniques to project the inventory balance each year for comparison with the client's assertion regarding the inventory balance in the financial statements. In 20X7, the auditor's sample size was 112 inventory items. In 20X8, the number of units in inventory, the tolerable misstatement, and the required confidence were the same as in 20X7; however, the population variability increased due to the introduction of a new product line. The 20X8 required sample size would, therefore, be: Larger than the 20X7 required sample size. The same as the required sample size in 20X7. Indeterminate based on the information provided. Smaller than the 20X7 required sample size.

Explanation Choice "1" is correct. Population variability has direct effect on sample size. Choices "4", "2", or "3" are incorrect, as population variability has direct effect on sample size.

Which of the following is a communication requirement with respect to registered audit firms? Registered firms must report to the audit committee the critical accounting policies and practices used. Registered firms must report to the internal audit manager any alternative accounting treatments discussed with management. Registered firms must provide to management a schedule of unadjusted audit differences. Registered firms must provide a copy of the firm's quality control policies and procedures to management and the audit committee.

Explanation Choice "1" is correct. Registered firms must report to the audit committee the critical accounting policies and practices used. Choice "2" is incorrect. Registered firms must report to the audit committee (not to the internal audit manager) any alternative accounting treatments discussed with management. Choice "3" is incorrect. Registered firms must provide to the audit committee (not to management) a schedule of unadjusted audit differences. Choice "4" is incorrect. There is no requirement that registered firms provide a copy of the firm's quality control policies and procedures to management and the audit committee. However, the registered firm is required to provide a written statement regarding their independence to the audit committee.

Which of the following subsequent events would most likely result in adjustment to the financial statements? After the balance sheet date and before the financial statements are issued, the company settles litigation that had resulted in a liability as of the balance sheet date. A strike occurred after year-end that may have a material effect on the client's future operations. After the balance sheet date and before the financial statements are issued, the company disposes of a segment of the business in a sale to an unrelated third party. After the balance sheet date and before the financial statements are issued, the company issues stock in the acquisition of a new business

Explanation Choice "1" is correct. Since the litigation existed at the balance sheet date, GAAP requires financial statement adjustment to reflect events that occurred after year-end, but before the financial statements were issued. Choice "2" is incorrect. The strike occurred after year-end, so it would not require financial statement adjustment, although it might require disclosure. Choice "3" is incorrect. Disposal of a business segment occurring after year-end does not require financial statement adjustment, although it might require disclosure. Choice "4" is incorrect. Stock issuances occurring after year-end do not require financial statement adjustment, although they might require disclosure.

An auditor notes that retained earnings includes an amount appropriated in accordance with a loan covenant. Which procedures would the auditor use to audit retained earnings? I. Trace dividend declarations to appropriate authorization by management. II. Analyze the retained earnings account since the last audit. III. Use a standard bank confirmation to confirm the amount appropriated. II only. I, II, and III. I and III, only. II and III, only.

Explanation Choice "1" is correct. Statement II is true. The auditor should analyze the retained earnings account since the last audit. Generally this is fairly easy to do, since there are not very many direct entries to retained earnings. Statement I is false because dividends should be approved by the board of directors, not by management. Statement III is false because appropriations represent restrictions on the amount that can be paid as dividends, not a segregation of actual cash funds, so the bank would be unable to confirm appropriation amounts. Choices "2", "3", and "4" are incorrect, based on the above explanation.

Lou, CPA, is reviewing the financial statements of Trading Corporation and becomes aware of a material departure from generally accepted accounting principles. What is the proper next course of action for Lou to take? Recommend that Trading Corporation revise the financial statements to conform to generally accepted accounting principles. Withdraw from the engagement and provide no further services. Modify the report by stating that, except for the issue noted in the Known Departure from Accounting Principles Generally Accepted in the United States of America, Lou is not aware of any material modifications that should be made. Modify the report by adding a separate paragraph disclosing the departure.

Explanation Choice "1" is correct. The accountant should recommend that the financial statements be revised to conform to generally accepted accounting principles. If the financial statements are not revised, then the accountant might consider modifying the report, or, if modification is inadequate, withdrawing from the engagement. Choices "4", "3", and "2" are incorrect, based on the above explanation.

Which of the following is not true about the five components of internal control? The auditor is required to classify relevant controls into one of the five component categories. The auditor is required to have an understanding of each component. The five components of internal control are applicable to all audits. Each of the five components of internal control may affect any of the three overall entity objectives.

Explanation Choice "1" is correct. The auditor is not required to classify relevant controls into one of the five component categories, but rather should use the component categories as a useful framework for identifying and evaluating controls. Choice "2" is incorrect. The auditor is required to obtain an understanding of each component of internal control sufficient to plan the audit. Choice "3" is incorrect. The five components of internal control are applicable to the audit of every entity. Choice "4" is incorrect. The five components of internal control may affect any of the three overall entity objectives (reliable financial reporting, effective/efficient operations, and compliance.)

In planning an audit of a new client, an auditor most likely would consider the methods used to process accounting information because such methods: Influence the design of internal control. Affect the auditor's preliminary judgment about materiality levels. Assist in evaluating the planned audit objectives. Determine the auditor's acceptable level of audit risk.

Explanation Choice "1" is correct. The auditor should consider the methods the entity uses to process accounting information in planning the audit because such methods influence the design of internal control. The extent to which computer processing is used in significant accounting applications, as well as the complexity of the processing, may also influence the nature, timing, and extent of audit procedures. Choice "2" is incorrect. Materiality is a matter of professional judgment and is influenced by the auditor's perceptions of the needs of a reasonable person. It would not be affected by the methods used to process accounting information. Choice "3" is incorrect. The auditor develops specific audit objectives based on financial statement assertions. The methods used to process accounting information would not be relevant to the development of objectives. Choice "4" is incorrect. Audit risk is the risk that the auditor may unknowingly fail to modify the opinion on financial statements that are materially misstated. The acceptable level of audit risk is a matter of auditor judgment, but it would not be affected by the methods used to process accounting information.

In a probability-proportional-to-size sample with a sampling interval of $5,000, an auditor discovered that a selected account receivable with a recorded amount of $10,000 had an audit amount of $8,000. If this were the only error discovered by the auditor, the projected error of this sample would be: $2,000 $1,000 $4,000 $5,000

Explanation Choice "1" is correct. There is a $2,000 projected error in this PPS sample. Recorded Amount − Audit Amount = Tainting Percentage × Sample Interval = Projected Error $10,000 − $8,000 = N/A N/A $2,000 N/A = not applicable, as the actual difference of $2,000 is used when the recorded amount is larger than the sample interval. Choices "2", "3", and "4" are incorrect, based on the above explanation.

In testing controls over cash disbursements, an auditor most likely would determine that the person who signs the check also: Stamps, perforates or otherwise cancels supporting documents. Returns the checks to accounts payable. Approves the voucher for payment. Is denied access to the supporting documents.

Explanation Choice "1" is correct. This prevents duplicate payments of vendor invoices since cancellation occurs immediately after the check is signed. Choice "2" is incorrect. Accounts payable should not get the signed checks, since this would allow one group to have both recordkeeping and custodial duties. The check signer should mail the check. Choice "3" is incorrect. The responsibility for authorization and custody should be separated. Choice "4" is incorrect. The person who signs the check should have access to the supporting documents, as they must be cancelled after payment.

Which of the following cash transfers results in a misstatement of cash at December 31, 20X7? Bank Transfer Schedule Disbursement recorded in books Deposit received in books Paid by bank Recorded received by bank 1/5/X8 12/31/X7 12/31/X7 1/4/X8 1/1/X8 1/1/X8 12/31/X7 12/31/X7 12/31/X7 12/30/X7 12/30/X7 1/4/X8 1/4/X8 1/7/X8 1/4/X8 1/4/X8

Explanation Choice "1" is correct. This transfer results in a misstatement of cash, because the payment was not recorded in the period of deposit. Choices "2", "4", and "3" are incorrect, because the disbursement and the deposit were recorded in the same period in each of these situations.

Pell, CPA, decides to serve as group engagement partner in the audit of the financial statements of Tech Consolidated, Inc. Smith, CPA, audits one of Tech's subsidiaries. In which situation(s) should Pell make reference to Smith's audit under U.S. GAAS? I. Pell reviews Smith's audit documentation and assumes responsibility for Smith's work, but expresses a qualified opinion on Tech's financial statements. II. Pell is unable to review Smith's audit documentation; however, Pell's inquiries indicate that Smith has an excellent reputation for professional competence and integrity. II only. I only. Both I and II. Neither I nor II.

Explanation Choice "1" is correct. Under U.S. GAAS, the group engagement partner makes reference in the audit report to the work of the component auditor when the group engagement partner is unable to review the component auditor's audit documentation. This is because the group engagement partner will be unable to be satisfied concerning the work performed by the component auditor. Even though the component auditor has an excellent reputation, the group engagement partner must see the work to be able to assume responsibility for it. Note that under ISAs, no reference is made to the component auditor unless required by law or regulation. Choice "2" is incorrect. When the group engagement partner decides to assume responsibility for the work of the component auditor, no reference is made to the work of the component auditor, regardless of the type of audit report expressed. Choice "3" is incorrect. When the group engagement partner decides to assume responsibility for the work of the component auditor, no reference is made to the work of the component auditor, regardless of the type of audit report expressed. Choice "4" is incorrect. The group engagement partner will make reference in the audit report to the work of the component auditor when the group engagement partner is unable to review the component auditor's audit documentation. This is because the group engagement auditor will be unable to be satisfied concerning the work performed by thecomponent auditor. Even though the component auditor has an excellent reputation, the group engagement partner must see the work to be able to assume responsibility for it.

When management does not provide reasonable justification that a change in accounting principle is preferable and it presents comparative financial statements, the auditor should express a qualified opinion: Each year that the financial statements initially reflecting the change are presented. Only in the year of the accounting principle change. Each year until management changes back to the accounting principle formerly used. Only if the change is to an accounting principle that is not generally accepted.When management does not provide reasonable justification that a change in accounting principle is preferable and it presents comparative financial statements, the auditor should express a qualified opinion: Each year that the financial statements initially reflecting the change are presented. Only in the year of the accounting principle change. Each year until management changes back to the accounting principle formerly used. Only if the change is to an accounting principle that is not generally accepted.

Explanation Choice "1" is correct. When management does not provide reasonable justification that a change in accounting principle is preferable and it presents comparative FS, the auditor should express a qualified opinion each year that the FS initially reflecting the change are presented.

A report includes the following language, "This report is intended solely for the information and use of the board of directors and management of X Company, and is not intended to be and should not be used by anyone other than the specified parties." This report would most likely relate to which of the following engagements? A report on an examination of a financial forecast. A report on a client's compliance with a regulatory requirement, assuming the report is prepared based on a financial statement audit of the complete financial statements. A report on a specified element in a financial statement, where that element is prepared in accordance with the tax basis of accounting. A report on financial statements prepared on the cash basis of accounting.

Explanation Choice "2" is correct. A report on a client's compliance with a regulatory requirement, assuming the report is prepared based on a financial statement audit of the complete financial statements, would contain restricted use language. Choice "1" is incorrect. A report on an examination of a financial forecast does not require a restriction on the use of the report. Choice "3" is incorrect. A report on a specified element in a financial statement, where that element is prepared in accordance with the tax basis of accounting, would not contain restricted use language. Choice "4" is incorrect. A report on financial statements prepared in conformity with the cash basis of accounting does not require a restriction on the use of the report.

Which of the following is true about appointing the external auditor after year-end? It is only permitted with the express agreement of management, the audit committee, and external auditor. It is permitted as long as the auditor has considered and discussed with the client any limitations that may be caused by the late appointment. It is typical in most cases. It allows the auditor to plan a more efficient audit because more of the data is known after the year has ended

Explanation Choice "2" is correct. An auditor is permitted to accept an engagement near or after yearend. The auditor should consider whether late appointment would pose limitations on the audit that may lead to a qualified opinion or a disclaimer of opinion, and should discuss such concerns with the client. Choice "1" is incorrect. There is no requirement that management agree to the timing of the independent auditor appointment. Choice "3" is incorrect. The auditor is typically appointed before year-end to allow for such things as a year-end inventory observation or securities count. Choice "4" is incorrect. Early appointment (not late appointment) of the auditor allows the auditor to plan a more efficient audit.

When issuing an unmodified opinion, the auditor who evaluates the audit findings should be satisfied that the: Amount of factual misstatements are acknowledged and recorded by the client. Estimate of the total misstatement is less than a material amount. Amount of identified misstatements are documented in the management representation letter. Estimate of the total misstatement includes the adjusting entries already recorded by the client.

Explanation Choice "2" is correct. An unmodified opinion states that the financial statements are presented fairly, in all material respects. Accordingly, if the auditor believes that total misstatement (includes factual, judgmental, and projected misstatements) is immaterial, an unmodified opinion is appropriate. Choice "1" is incorrect. Even if the client adjusts the financial statements to correct factual misstatements, the auditor still must feel comfortable that any remaining misstatements would not be material to the financial statements before rendering an unmodified opinion. Choice "3" is incorrect. Documenting misstatements in the management representation letter is not required, nor would it eliminate the need to modify the opinion if the identified misstatements were material. Choice "4" is incorrect. An auditor is not concerned with misstatements already corrected by the client. It is the auditor's estimate of uncorrected--and perhaps unknown--errors that affects the type of opinion rendered.

Audit evidence is obtained by the auditor when performing all of the following, except when: Completing a risk assessment. Determining the sample size. Performing substantive procedures. Reviewing the previous audit.

Explanation Choice "2" is correct. Audit evidence is not gathered when the auditor is determining the sample size. After the sample size is selected, the auditor will gather audit evidence by performing substantive procedures, test of controls, and other audit procedures. Choice "1" is incorrect. The auditor can obtain audit evidence while performing risk assessment procedures. Choice "3" is incorrect. Performing substantive procedures is a primary means to gather audit evidence, along with test of controls. Choice "4" is incorrect. Audit evidence can be gathered for the current audit by reviewing work papers from the previous audit.

As the acceptable level of detection risk decreases, an auditor may change the: Assessed level of inherent risk. Nature of substantive tests from a less effective to a more effective procedure. Timing of substantive tests from year-end to an interim date. Assessed level of control risk from high to low.

Explanation Choice "2" is correct. Better evidence must be obtained to achieve a lower level of detection risk. Choice "1" is incorrect. The assessed level of inherent risk (based on the auditor's evaluation of the nature of the assertion) determines the risk of material misstatement, which in turn affects the acceptable level of detection risk. The acceptable level of detection risk is therefore determined by the assessed level of inherent risk, not vice versa. Choice "3" is incorrect. Shifting tests from year-end to interim increases detection risk. Choice "4" is incorrect. The assessed level of control risk (based on the auditor's evaluation of the client's controls) determines the risk of material misstatement, which in turn affects the acceptable level of detection risk. The acceptable level of detection risk is therefore determined by the assessed level of control risk, not vice versa.

The risk of incorrect rejection and the likelihood of assessing control risk too high relate to the: Preliminary estimates of materiality levels. Efficiency of the audit. Allowable risk of tolerable misstatement. Effectiveness of the audit

Explanation Choice "2" is correct. Both incorrect rejection and assessing control risk too high will cause the auditor to perform more procedures than are necessary, which affects audit efficiency. Choice "1" is incorrect. The risk of incorrect rejection and the likelihood of assessing control risk too high are both part of sampling risk, which is unrelated to preliminary materiality levels. Choice "3" is incorrect. The risk of incorrect rejection and the likelihood of assessing control risk too high are both part of sampling risk, which is unrelated to tolerable misstatement. (Note also that there is no defined term "allowable risk of tolerable misstatement.") Choice "4" is incorrect. The effectiveness of the audit is not affected by these risks, as the auditor will perform enough additional testing to eventually come to the proper conclusion.

Reporting on internal control to meet the requirements of Government Auditing Standards differs from reporting under other generally accepted auditing standards in that Government Auditing Standards requires a: Statement of negative assurance that internal controls not tested have an immaterial effect on the entity's financial statements. Report describing the scope of the auditor's testing of compliance and of internal control. Statement of positive assurance that internal controls designed to detect material errors and fraud were tested. Written report describing the entity's internal controls specifically designed to prevent fraud, abuse, and illegal acts.

Explanation Choice "2" is correct. Government Auditing Standards require a description of the scope of the auditor's testing of compliance and of internal control. This is not required under generally accepted auditing standards. Choice "1" is incorrect. Government Auditing Standards require tests of compliance with applicable laws and regulations governing a particular grant, because the grant money received can only be spent for certain purposes. However, neither GAGAS nor GAAS requires that negative assurance be given with respect to internal controls. Choice "3" is incorrect. Although Government Auditing Standards require tests of compliance with applicable laws and regulations governing a particular grant (the grant money received can only be spent for certain purposes), auditors are not required to give positive assurance that internal controls designed to detect material errors and fraud were tested under either GAGAS or GAAS. Choice "4" is incorrect. Although a written report on internal control is required, the auditor does not have to identify procedures specifically designed to prevent fraud, abuse, and illegal acts.

Whitney, CPA, is assessing the auditability of Nissen Manufacturing, a possible new client. Which of the following would be least likely to cause Whitney to reject Nissen as a new client? Management of Nissen expresses a disregard for maintaining an adequate internal control environment. Management of Nissen is unwilling to send accounts receivable confirmations, due to a desire not to trouble customers. Management of Nissen is unable to provide financial records for the second half of the year due to a computer malfunction. Management of Nissen is unwilling to provide all financial records to Whitney, due to a desire to keep such information confidential.

Explanation Choice "2" is correct. If a potential client is unwilling to send accounts receivable confirmations, the auditor may perform alternative procedures, such as reviewing subsequent cash receipts. Choice "1" is incorrect. Management's disregard for its responsibility to maintain an adequate internal control environment compromises its ability to provide reasonable assurance regarding reliable financial reporting and may lead the auditor to decide not to accept a new engagement because the risk of financial statement misstatement is too high. Choices "3" and "4" are incorrect. If a potential client is unable or unwilling to provide the financial information needed by the auditor to complete the audit, the auditor may not be willing to accept the engagement.

An auditor most likely would apply analytical procedures in the overall review stage of an audit to: Evaluate the effectiveness of the internal control activities. Determine whether additional audit evidence may be needed. Identify auditing procedures omitted by the staff accountants. Enhance the auditor's understanding of subsequent events

Explanation Choice "2" is correct. In performing analytical procedures as an overall review, the auditor determines whether adequate evidence has been gathered in response to unusual or unexpected balances identified during the audit, and may decide that additional audit procedures are warranted. In addition, the auditor may identify unusual or unexpected balances not already noted during the audit, which would also require the application of further auditing procedures. Choice "1" is incorrect. Analytical procedures applied during the overall review stage of the audit are meant to evaluate the overall financial statement presentation, and to assess the conclusions reached by the auditor. This is a high-level review, and one that focuses on the financial statements. As such, it would not be useful in evaluating the effectiveness of the client's internal control activities. Choice "3" is incorrect. Analytical procedures applied during the overall review stage of the audit are meant to evaluate the overall financial statement presentation, and to assess the conclusions reached by the auditor. This is a high-level review, and one that focuses on the financial statements. As such, it would not be likely to identify omitted auditing procedures. Choice "4" is incorrect. Analytical procedures applied during the overall review stage of the audit are meant to evaluate the overall financial statement presentation, and to assess the conclusions reached by the auditor. This is a high-level review, and one that focuses on the financial statements. As such, it would not be likely to enhance the auditor's understanding of subsequent events.

Which of the following audit procedures, if used, should be combined with other audit procedures when testing the operating effectiveness of controls? Observation. Inquiry. Inspection. Reperformance.

Explanation Choice "2" is correct. Inquiry should be corroborated with other audit procedures when testing the operating effectiveness of controls. Inquiry alone is not sufficient to support a conclusion on the operating effectiveness of controls. Choice "1" is incorrect. Observation does not need to be combined with other audit procedures when testing operating effectiveness of controls. Choice "3" is incorrect. Inspection does not need to be combined with other audit procedures when testing operating effectiveness of controls. Choice "4" is incorrect. Reperformance does not need to be combined with other audit procedures when testing operating effectiveness of controls

Which of the following ratios would most likely be used to evaluate an entity's profitability? Dividends per common share / earnings per share Net income / net sales Cost of goods sold / average inventory Net sales / average total assets

Explanation Choice "2" is correct. Net income divided by net sales, which equals net profit margin, indicates the percentage of every sales dollar that becomes profit. Choice "1" is incorrect. Dividends per common share divided by earnings per share measures the percentage of earnings being paid to shareholders, but it does not indicate the company's overall profitability. Choice "3" is incorrect. Cost of goods sold divided by average inventory measures how quickly inventory is sold. Choice "4" is incorrect. Net sales divided by average total assets measures how effectively assets are used.

The accountant may report on agreed-upon procedures applied to specified elements, accounts, or items of financial statements: But must be independent of the client company and provide limited assurance on the sufficiency of the agreed-upon procedures performed. But must be independent of the client company and give no opinion or any other form of assurance on the sufficiency of the agreed-upon procedures applied. Without being independent of the client company since no opinion or any other form of assurance is given on the sufficiency of agreed-upon procedures performed. As long as the accountant assumes responsibility for the adequacy of the agreedupon procedures the accountant has agreed to perform.

Explanation Choice "2" is correct. The accountant must be independent to perform an agreed-upon procedures engagement, and should not provide an opinion or any other form of assurance on the sufficiency of the procedures applied. The specified party (often the client) is responsible for the sufficiency of the procedures. Choice "1" is incorrect. The accountant should not provide an opinion or any other form of assurance on the sufficiency of the procedures applied. The specified party (often the client) is responsible for the sufficiency of the procedures. Choice "3" is incorrect. The accountant must be independent to perform an agreed-upon procedures engagement. Choice "4" is incorrect. The specified party (often the client) is responsible for the adequacy of the procedures.

Which of the following representations does an auditor make explicitly and which implicitly when expressing an unmodified opinion? Consistent Application of GAAP In accordance with GAAP in the U.S.A. Implicitly Implicitly Implicitly Explicitly Explicitly Implicitly Explicitly Explicitly

Explanation Choice "2" is correct. The audit report specifically states whether the financial statements are presented in accordance with accounting principles generally accepted in the United States of America. On the other hand, consistency is not mentioned in the report, but is implied (i.e., it is assumed that there is consistency when there is no mention of it in the report). Choice "1" is incorrect. The audit report specifically states whether the financial statements are presented in accordance with accounting principles generally accepted in the United States of America. Choice "3" is incorrect. The audit report specifically states whether the financial statements are presented in accordance with accounting principles generally accepted in the United States of America. On the other hand, consistency is not mentioned in the report, but is implied (i.e., it is assumed that there is consistency when there is no mention of it in the report). Choice "4" is incorrect. Consistency is not mentioned in the report, but is implied (i.e., it is assumed that there is consistency when there is no mention of it in the report).

Which is true about accounts receivable confirmations? Confirmations provide evidence about existence, rights, and valuation. Confirmations may be based on single transactions rather than entire customer balances. The client may mail the confirmations, but they should be returned directly to the auditor. Blank confirmations provide improved response rates but may be less reliable.

Explanation Choice "2" is correct. The auditor should consider the types of information respondents will be readily able to confirm. For example, it may be easier for certain respondents to confirm individual transactions rather than entire balances. In such cases, the auditor may choose to confirm based on invoice number (i.e., single transactions). Choice "1" is incorrect. Accounts receivable confirmations do not provide reliable evidence about valuation. Just because a respondent confirms that he/she owes a certain amount does not mean that he/she has the intent or the ability to pay it. Choice "3" is incorrect. The auditor should maintain control over the confirmation requests and responses, which means the auditor should be the one to mail the confirmations, not the client. Choice "4" is incorrect. Blank confirmations are those in which the recipient is requested to fill in the balance. They provide improved reliability since the respondent cannot simply sign off without actually verifying the balance. However, since more work is required, response rates are often decreased.

The management of Cain Company, a nonissuer, engaged Bell, CPA, to audit Cain's internal control. Bell's report described several material weaknesses and potential errors and irregularities that could occur. Subsequently, management included Bell's report in its annual report to the Board of Directors with a statement that the cost of correcting the weaknesses would exceed the benefits. Bell should: Advise management that Bell's report was restricted for use only by management. Disclaim an opinion as to management's cost-benefit statement. Advise the Board that Bell either agrees or disagrees with management's statement. Advise both management and the Board that Bell was withdrawing the opinion.

Explanation Choice "2" is correct. The auditor should disclaim an opinion as to management's costbenefit statement (i.e., "We do not express an opinion or any other form of assurance on management's cost-benefit statement."). Choice "1" is incorrect. The CPA's report on internal control is not restricted as to use. Choice "3" is incorrect. The CPA should disclaim an opinion regarding management's representation. Choice "4" is incorrect. The CPA does not need to withdraw the opinion as long as a disclaimer on management's cost-benefit statement is presented.

The purpose of segregating the duties of hiring personnel and distributing payroll checks is to separate the: Administrative function from the hiring function. Authorization of transactions from the custody of related assets. Human resources functions from the accounting functions. Application controls from the general controls.

Explanation Choice "2" is correct. The hiring function provides authorization for payment. Distributing payroll is a custodial function. Choice "1" is incorrect. Hiring is an administrative function. Choice "3" is incorrect. Segregation of duties generally refers to the separation of authorization, recordkeeping and custody, not the separation of various functional units. Choice "4" is incorrect. Hiring and payroll distribution are not types of application controls or general controls.

Before issuing an unmodified report on a compliance audit, an auditor becomes aware of an instance of material noncompliance occurring after the period covered by the audit. The least appropriate response by the auditor would be to: Determine whether the noncompliance relates to conditions that existed as of period end or arose subsequent to the reporting period. Issue a qualified compliance report describing the subsequent noncompliance. Discuss the matter with management and, if appropriate, those charged with governance. Modify the standard compliance report to include a paragraph describing the nature of the subsequent noncompliance.

Explanation Choice "2" is correct. The least appropriate response if an auditor discovers an instance of material noncompliance that occurred after the period covered by the audit is to issue a qualified compliance report describing the subsequent noncompliance. The opinion issued (i.e., unmodified) should relate to compliance during the period covered by the auditor's report. However, if the auditor determines that it is of such nature and significance that its disclosure is needed to prevent report users from being misled, the auditor may add an other-matter paragraph in the report to describe the nature of the noncompliance. Choice "1" is incorrect. An appropriate response is to determine whether the noncompliance relates to conditions that existed as of period end or arose subsequent to the reporting period.

When assessing the competence of an entity's internal auditor, an independent CPA should obtain information about all of the following except: Quality of the internal auditor's audit documentation. The organizational status of the internal auditor. Professional certification of the internal auditor. Education level and professional experience of the internal auditor.

Explanation Choice "2" is correct. The organizational status of the internal auditor is used to evaluate objectivity. Choice "1" is incorrect. An internal auditor's competence may be evaluated based upon the quality of his or her audit documentation. Choice "3" is incorrect. An internal auditor's competence may be evaluated based upon his or her professional certification. Choice "4" is incorrect. An internal auditor's competence may be evaluated based upon his or her educational level and professional experience.

When auditing a client's related party transactions (relationships), certain audit objectives should be met. Which of the following does not represent a primary audit objective pertaining to related party transactions? Determine whether the client's financial statements achieve fair presentation of all related party transactions and relationships. Determine which of the client's related party transactions were not completed on an arm's length transactions basis. Recognize fraud risk factors arising from the client's related party transactions. Obtain sufficient audit evidence that the client's related party transactions have been identified and disclosed.

Explanation Choice "2" is correct. This is not a primary audit objective. By nature, related party transactions are not considered an arm's length transaction due to the related parties involved and the potential for preferential terms. Additionally, with the exception of minor or routine transactions, it is often difficult for an auditor to determine whether a given related party transaction took place in the same manner as an unrelated party transaction. Choices "1", "4", and "3" are incorrect as each represents a primary audit objective when auditing related party transactions.

An auditor is conducting an attribute sampling application to test the effectiveness of a particular control. The following rates have been determined: Sample deviation rate: 5% Tolerable deviation rate: 6% Allowance for sampling risk: 2% Which of the following is true? Since the sample deviation rate exceeds the tolerable deviation rate less the allowance for sampling risk, the auditor may rely on the control. Since the tolerable deviation rate plus the allowance for sampling risk exceeds the sample deviation rate, the auditor may not rely on the control. Since the tolerable deviation rate less the allowance for sampling risk is less than the sample deviation rate, the auditor may not rely on the control. Since the tolerable deviation rate exceeds the sample deviation rate, the auditor may rely on the control.

Explanation Choice "3" is correct. A control cannot be relied upon when: sample deviation rate (S) + allowance for sampling risk (A) > = tolerable deviation rate (T) This can be restated as S > = T − A , which is what this option describes. Choice "1" is incorrect. This option states that S > = T − A, which is true; however, it goes on to conclude that the control may be relied upon, which is false. Choice "2" is incorrect. The sum of the tolerable deviation rate plus the allowance for sampling risk (T + A) is not a part of the proper equation, which compares S + A to T, nor can an algebraic manipulation be used to restate it in this format. Choice "4" is incorrect. The sample deviation rate should not be compared to the tolerable deviation rate without an adjustment for sampling risk.

A practitioner's report on agreed-upon procedures should contain which of the following statements? Sufficiency of procedures is the responsibility of the practitioner. All classification codes appeared to comply with such performance documents. The procedures performed were those agreed to by the specified parties identified in the report. Nothing came to my attention as a result of applying the procedures.

Explanation Choice "3" is correct. A practitioner's report on agreed-upon procedures should contain the statement that the procedures performed were those agreed to by the specified parties identified in the report. Choice "1" is incorrect. A practitioner's report on agreed-upon procedures should contain the statement that the sufficiency of procedures is the responsibility of the specified parties, not the practitioner. The practitioner disclaims responsibility for the sufficiency of the procedures. Choice "2" is incorrect. A practitioner's report on agreed-upon procedures should not include the statement that all classification codes appeared to comply with such performance documents. The agreed upon procedures engagement may relate to a subject matter unrelated to classification of codes, and an agreed-upon procedures report does not include any form of assurance. Choice "4" is incorrect. A practitioner's report on agreed-upon procedures contains a disclaimer of opinion on the subject matter. Limited assurance is not provided in an agreedupon procedures engagement

A special report on financial statements prepared on the cash basis of accounting should include: A disclaimer of opinion, because an auditor should not report on financial statements that are not designed to be in conformity with GAAP. An emphasis-of-matter paragraph including a brief explanation of the cash basis of accounting. A statement that the audit was conducted in accordance with generally accepted auditing standards. A qualified or adverse opinion, due to the departure from GAAP.

Explanation Choice "3" is correct. A special report on financial statements prepared on the cash basis of accounting should include, in the auditor's responsibility paragraph, a statement that the audit was conducted in accordance with generally accepted auditing standards. Choice "1" is incorrect. A special report on financial statements prepared on the cash basis of accounting includes positive assurance, not a disclaimer of opinion, regarding whether the financial statements are presented fairly in conformity with the cash basis of accounting. Choice "2" is incorrect. A special report on financial statements prepared on the cash basis of accounting does include an emphasis-of-matter paragraph, but it does not provide an explanation of the cash basis of accounting. Instead, it refers to the note that discusses the cash basis of accounting. Choice "4" is incorrect. A special report on financial statements prepared on the cash basis of accounting does not require a qualified or adverse opinion. An "unmodified" opinion may be presented stating that the financial statements are presented fairly in conformity with the cash basis of accounting.

Which of the following procedures would an auditor most likely perform to identify unusual sales transactions? Tracing cash receipt entries to the bank statement deposit for amount and date. Examining duplicate sales invoices for credit approval by the credit manager. Performing a trend analysis of quarterly sales. Tracing credits in the accounts receivable ledger to source documentation.

Explanation Choice "3" is correct. An auditor is most likely to perform a trend analysis of quarterly sales to identify unusual sales transactions. This is considered an analytical procedure. Choice "1" is incorrect. Tracing cash receipt entries to the bank statement deposit for amount and date may help to detect lapping, which is related to theft of cash, rather than detect unusual sales transactions. Choice "2" is incorrect. Examining duplicate sales invoices for credit approval by the credit manager would provide evidence that sales on account were appropriately authorized, but would unlikely help the auditor identify unusual sales transactions. Choice "4" is incorrect. Tracing credits in the accounts receivable ledger to source documentation is unlikely to help identify unusual sales transactions. Credits to accounts receivable would likely be indicative of payment on account or a return of sale.

Which of the following are required as part of an auditor's planning process? Understanding the design of controls Determining whether controls have been implemented Evaluating the operating effectiveness of controls Documenting the understanding of internal control Yes No No Yes No Yes Yes No Yes Yes No Yes No No Yes No

Explanation Choice "3" is correct. As part of planning, the auditor is required to obtain an understanding of the design of controls and determine whether they have been implemented, as well as to document this understanding. The auditor is not required to evaluate the operating effectiveness of controls during the planning process. Choices "1", "4", and "2" are incorrect, based on the above explanation.

Which of the following are elements of a firm's quality control that should be considered in establishing its quality control policies and procedures? Assigning personnel, client acceptance, and analytical review. Client acceptance, professional development, and analytical review. Assigning personnel, client acceptance, and professional development. Assigning personnel, professional development, and analytical review.

Explanation Choice "3" is correct. Assigning personnel to engagements and providing for professional development are part of the human resources element of quality control. Engagement/client acceptance and continuance is another element of quality control. Choices "2", "4", or "1" are incorrect. Analytical review procedures are not an element of a firm's system of quality control.

An auditor is engaged to report on the fairness of the financial statements of ABC Corp., a nonissuer. The auditor identifies several material misstatements in sales, which were caused by a control deficiency in the sales orders' preparation process. This was the only control deficiency and the only material misstatement identified in testing. The auditor presented a journal entry to the client to correct these misstatements, which the client agreed to record. Which of the following actions is the auditor least likely to perform? Determine whether the deficiency represents a significant deficiency or a material weakness. Reassess control risk. Issue a qualified opinion on the financial statements. Consider compensating controls around the sales orders' preparation process.

Explanation Choice "3" is correct. Because the client agreed to adjust its financial statements, the company should receive an unmodified opinion. Note: It is possible for a company to have a material weakness in internal control but still receive an unmodified opinion on the fairness of the financial statements. Choice "1" is incorrect. When control deficiencies are noted, the auditor should determine whether the control deficiency is a significant deficiency or material weakness. Choice "2" is incorrect. Identification of control deficiencies may result in the auditor reassessing control risk (i.e., from low to high.) Choice "4" is incorrect. When control deficiencies are noted, the auditor should consider whether compensation controls are present.

Before accepting an audit engagement, a successor auditor should make specific inquiries of the predecessor auditor regarding the predecessor's: Opinion of any subsequent events occurring since the predecessor's audit report was issued. Awareness of the consistency in the application of GAAP between periods. Understanding as to the reasons for the change of auditors. Evaluation of all matters of continuing accounting significance.

Explanation Choice "3" is correct. Before accepting an engagement, the auditor should make specific inquiries of the predecessor auditor in order to assist the auditor in deciding whether or not to accept the engagement. Inquiry should include the predecessor's understanding of the reasons for the change in auditors. Choice "1" is incorrect. The predecessor generally does not provide an opinion on events occurring subsequent to the issuance of the audit report. Choice "2" is incorrect. The successor (and not the predecessor) evaluates the consistency in the application of GAAP. This evaluation occurs after acceptance. Choice "4" is incorrect. The predecessor generally allows the successor to review audit documentation related to matters of continuing accounting significance, but this occurs subsequent to acceptance.

Which of the following circumstances requires modification of the accountant's report on a review of interim financial information of a publicly held entity prepared on the basis of generally accepted accounting principles? An uncertainty, lack of adherence to generally accepted accounting principles, and lack of adequate disclosure. None of the following: an uncertainty, an accounting change, lack of adherence to generally accepted accounting principles, lack of adequate disclosure. Lack of adherence to generally accepted accounting principles and lack of adequate disclosure. An accounting change, lack of adequate disclosure, and lack of adherence to generally accepted accounting principles.

Explanation Choice "3" is correct. Departures from GAAP (including inadequate disclosure) require modification of the review report. Choice "1" is incorrect. Uncertainties and accounting changes do not require report modifications, as long as disclosure is adequate. Choice "2" is incorrect. Departures from GAAP (including inadequate disclosure) require modification of the review report. Choice "4" is incorrect. Accounting changes do not require report modifications, as long as disclosure is adequate.

At the conclusion of an audit, an auditor is reviewing the evidence gathered in support of the financial statements. With regard to the valuation of inventory, the auditor concludes that the evidence obtained is not sufficient to support management's representations. Which of the following actions is the auditor most likely to take? Consult with the audit committee and issue a disclaimer of opinion. Consult with the audit committee and issue a qualified opinion. Obtain additional evidence regarding the valuation of inventory. Obtain a statement from management supporting their inventory valuation

Explanation Choice "3" is correct. If an auditor has doubts about a material assertion (such as the valuation of inventory), he/she should gather sufficient evidence to eliminate the doubt. Choices "1" and "2" are incorrect. The auditor would not consult with the audit committee regarding the sufficiency of audit evidence obtained, as this is determined based on the auditor's own judgment. In addition, if the auditor is able to obtain additional evidence, it might be possible to issue an unmodified opinion. Finally, even if no additional evidence is available, the auditor will still need to decide whether a qualified opinion or a disclaimer of opinion is more appropriate, depending on materiality. Choice "4" is incorrect. Since management representations are in fact "statements from management," obtaining additional statements from management would not provide additional support.

Plinth Industries, an issuer, employs several former employees of its auditor, Lawrence & Whitney, CPAs. Adams, CEO, began his career with L&W, but left ten years ago; Brooks, CFO, was with L&W until 18 months ago, and James, internal auditor, left L&W six months prior to the start of the current year's audit. Which of the following properly describes the effect of these potential conflicts of interest? L&W may not audit Plinth because the CEO and CFO are both former L&W employees. L&W may not audit Plinth due to the employment of Brooks within the two-year period preceding the audit. L&W may audit Plinth. L&W may not audit Plinth due to the employment of James within the one-year period preceding the audit.

Explanation Choice "3" is correct. L&W may audit Plinth since none of the situations described presents a conflict of interest as described in SOX Title II. Choices "1" and "2" are incorrect. Neither Adams nor Brooks was employed by L&W within the one-year period preceding the audit, so neither position poses a conflict of interest as described in SOX Title II. Choice "4" is incorrect. The conflict of interest provisions of SOX Title II apply only to the issuer's CEO, CFO, Controller, or Chief Accounting Officer (or to any person serving in an equivalent position), not to an internal auditor.

During the planning stage of an audit, the auditor initially assessed both inherent risk and control risk at a high level. Further testing of the client's internal controls led the auditor to reduce the assessment of control risk. Which of the following will most likely occur as a result? The auditor may decrease the allowed level of detection risk. The auditor may rely solely on analytical procedures, with no substantive procedures performed. The auditor may reduce the amount of substantive procedures performed. The auditor may reduce the assessment of inherent risk to match the control risk, since they were assessed at the same level during the initial planning.

Explanation Choice "3" is correct. Lower control risk reduces the total risk of material misstatement. When the risk of material misstatement decreases, the auditor may choose to reduce the amount of substantive procedures performed, although substantive procedures cannot be eliminated entirely. Choice "1" is incorrect. A reduction in control risk, all other things remaining the same, would lead to an increase in the allowed level of detection risk, not a decrease. Choice "2" is incorrect. An auditor may not entirely eliminate substantive procedures, regardless of the level of detection risk or risk of material misstatement. Choice "4" is incorrect. The assessment of control risk is independent from the auditor's assessment of inherent risk.

Nissen, CPA, is a registered public accounting firm. Which situation is least likely to violate PCAOB independence standards? Nissen provides tax services related to an aggressive tax transaction. Nissen provides tax services to the CEO of an audit client. Nissen provides non-audit services related to internal control to an audit client. Nissen accepts an engagement in which the engagement fee is dependent upon the results of the audit.

Explanation Choice "3" is correct. Non-audit services related to internal control must be communicated to the audit committee in writing, but they are not prohibited. The potential effects of the services should be discussed with the audit committee. Choice "1" is incorrect. Registered public accounting firms may not provide to audit clients any tax services related to certain aggressive tax transactions. Choice "2" is incorrect. Registered public accounting firms may not provide any tax services to corporate officers of the audit client. Choice "4" is incorrect. An engagement in which the fee is dependent upon the results of the audit (a contingent fee) is prohibited by PCAOB independence standards.

Which of the following audit techniques most likely would provide an auditor with the most assurance about the effectiveness of the operation of internal control? Inquiry of client personnel. Confirmation with outside parties. Observation of client personnel. Recomputation of account balance amounts

Explanation Choice "3" is correct. Observation of client personnel is the audit technique most likely to provide an auditor with the most assurance about the effectiveness of the operation of an internal control. This technique is especially effective for controls that leave little or no audit trail. Choice "1" is incorrect. While inquiry of client personnel can provide some evidence about the effectiveness of the operation of internal control, the auditor's direct observation generally provides greater assurance than does inquiry. Choice "2" is incorrect. Confirmation with outside parties is a common substantive test (direct test of an account balance), not a procedure used to test controls. Choice "4" is incorrect. Recomputation of account balance amounts is a substantive test, not a test of controls

An auditor would express an unmodified opinion with an emphasis-of-matter paragraph added to the report for: A justified change in accounting principle An unjustified change in accounting principle A justified change in accounting estimate No No Yes No Yes Yes Yes No No Yes Yes No

Explanation Choice "3" is correct. Only a justified change in accounting principle would result in an unmodified opinion with an emphasis-of-matter paragraph. An unjustified change leads to a qualified or adverse opinion, and a change in estimate does not require an emphasis-ofmatter paragraph. Choices "2", "4", and "1" are incorrect, based on the above explanation.

Which of the following documents should be sent to the accounts payable department? Purchase Order Sales Order Signed Check No Yes No No Yes Yes Yes No No Yes No Yes

Explanation Choice "3" is correct. The accounts payable department should receive the purchase order, receiving report, and vendor invoice, to assure that the goods received agree with what was ordered, and that the company is being billed only for what was received. Sales orders relate to the revenue cycle, and accounts payable would have no use for them. The treasurer should mail the signed check after it is signed; to send it to accounts payable (a recordkeeping department) would be an inappropriate segregation of duties. Choices "1", "2", and "4" are incorrect, based on the above explanation.

Management's written representation to the auditor in connection with a governmental audit would most likely include: Representation that all known noncompliance had been reported or negative assurance that other noncompliance likely does not exist. Negative assurance that the government has complied with compliance requirements. A statement that management has disclosed any communications from grantors concerning possible noncompliance. A statement that management had identified and disclosed all material government programs to the auditor.

Explanation Choice "3" is correct. The management letter will include a statement that management has disclosed any communications from grantors concerning possible noncompliance. Choice "1" is incorrect. Management will assert that they have disclosed all known noncompliance or positively state that there was no such noncompliance. Management representations do not provide negative assurance. Choice "2" is incorrect. The representation letter should include a statement that management believes that the entity has complied with compliance requirements. Management representations do not provide negative assurance. Choice "4" is incorrect. The representation letter should include a statement that management has disclosed all governmental programs to the auditor. Management's representation is not limited to only material government programs

The auditor performs a preliminary risk assessment for a new client. During the initial fieldwork phase the auditor's test of the client's controls indicate that certain controls are not operating effectively. Under this scenario, the auditor would most likely take which of the following actions? Perform additional audit procedures but do not revise the preliminary risk assessment. Issue a modified audit report. Revise the preliminary risk assessment and modify the planned audit procedures. Determine the impact on the client's financial statements without performing additional audit procedures.

Explanation Choice "3" is correct. Upon determining that the client's internal controls are not operating effectively, the auditor would then modify the preliminary risk assessment and modify the planned audit procedures. This is part of the auditor's ongoing assessment of risk during an audit. Choice "1" is incorrect. Although the auditor may perform further audit procedures under this scenario, the preliminary risk assessment would first be revised to make that determination. Choice "2" is incorrect. This is too severe of an action and, based on a revised risk assessment and additional audit procedures, may not be warranted. Choice "4" is incorrect. While ultimately the auditor will determine if the control weaknesses will result in a material misstatement of the client's financial statements, the risk assessment should first be revised and the planned audit procedures should be modified.

Which of the following is least likely to be used as a substantive test relating to cash balances? Obtain cutoff bank statements and perform bank reconciliations. Count all cash on hand. Verify that cash disbursements have been properly approved. Send cash confirmations to all banks with whom the client has done business.

Explanation Choice "3" is correct. Verifying that cash disbursements have been properly approved is a test of controls, not a substantive test. Choices "4", "2", and "1" are incorrect. Cash confirmations, cash counts, and bank reconciliations are all a means of verifying the ending cash balance.

An auditor may reasonably issue an "except for" qualified opinion for a(an): Scope limitation Unjustified accounting change No No Yes No Yes Yes No Yes

Explanation Choice "3" is correct. Yes - Yes. An "except for" qualified opinion is expressed when the "exceptions to GAAP" or scope restrictions are material but not pervasive. Choices "2", "4", and "1" are incorrect, based on the rule above.

Information gathered in the course of an independent audit is the property of the auditor. This information is not generally disclosed to outside parties. However, after discussion with legal counsel, the auditor may wish to disclose information about irregularities or noncompliance with laws and regulations to outside parties in which of the following circumstances? Change of auditor, court order, and accounting changes. Predecessor/successor communications, court order, and accounting changes. Change of auditor, predecessor/successor communications, and accounting changes. Change of auditor, predecessor/successor communications, and court order.

Explanation Choice "4" is correct. A duty to disclose such information outside the entity may exist when there is a change of auditor (reported to the SEC on Form 8-K), in response to a court order, and (with client permission) in response to a successor auditor's inquiries. Choices "3", "2", or "1" are incorrect. Accounting changes do not require disclosure to outside parties.

Which of the following questions would most likely be included in an internal control questionnaire concerning the completeness assertion for purchases? Is an authorized purchase order required before the receiving department can accept a shipment or the vouchers payable department can record a voucher? Are purchase requisitions prenumbered and independently matched with vendor invoices? Is the unpaid voucher file periodically reconciled with inventory records by an employee who does not have access to purchase requisitions? Are purchase orders, receiving reports, and vouchers prenumbered and periodically accounted for?

Explanation Choice "4" is correct. A question related to whether purchase orders, receiving reports and vouchers are prenumbered and periodically accounted for would most likely be included in an internal control questionnaire concerning the completeness assertion for purchases. A gap in recorded purchase order numbers might indicate an unrecorded purchase. Choice "1" is incorrect. Requiring an authorized purchase order before accepting a shipment would relate to whether the purchase was valid, not whether it was properly accounted for. Choice "2" is incorrect. Having prenumbered purchase requisitions independently matched with vendor invoices does not indicate whether all purchases are accounted for since these documents do not show that the purchase has been recorded. Choice "3" is incorrect. Reconciling the unpaid voucher file with inventory records does not indicate whether all purchases are accounted for since these documents do not show that all purchases have been recorded.

Which of the following procedures most likely would not be included in a review engagement of a nonissuer? Considering whether the financial statements conform with GAAP. Inquiring about subsequent events. Obtaining a management representation letter. Assessing control risk.

Explanation Choice "4" is correct. An accountant is not required to assess control risk as part of a review engagement. Assessment of control risk would be appropriate for an audit engagement. Choice "1" is incorrect. Considering whether the financial statements are in accordance with GAAP is an appropriate review procedure. Choice "2" is incorrect. Inquiring of management regarding subsequent events is an appropriate review procedure. Choice "3" is incorrect. Obtaining a management representation letter is an appropriate review procedure.

Which of the following is the most likely sequence of events an auditor might follow in considering internal control? An auditor performs additional tests of controls after the detailed substantive testing has been completed, because it may be more efficient to do so. An auditor performs substantive tests prior to tests of controls, because a large number of substantive errors may help the auditor identify areas of weakness. An auditor performs tests of controls first, using the knowledge obtained from those tests to develop an understanding of internal control. An auditor performs tests of controls at the same time that he or she is obtaining an understanding of internal control, because it may be more efficient to do so.

Explanation Choice "4" is correct. An auditor may choose to perform tests of controls at the same time that he or she is obtaining an understanding of internal control, because it may be more efficient to do so. Choice "1" is incorrect. Tests of controls are generally performed before substantive testing, as the assessed level of control risk is used to determine the nature, timing, and extent of substantive tests. Choice "2" is incorrect. The auditor uses his or her understanding of internal control, as well as the results of any tests of controls, to determine the nature, timing, and extent of substantive tests. Choice "3" is incorrect. The auditor generally would obtain an understanding of how internal control works first, to better be able to design appropriate tests of controls. While it is also acceptable to perform some tests of controls concurrently with obtaining an understanding of internal control, test of controls would not be performed prior to obtaining that understanding.

A letter issued on significant deficiencies relating to an entity's internal control observed during an audit of the financial statements of a nonissuer should include a: Paragraph describing management's evaluation of the effectiveness of internal control. Statement of compliance with applicable laws and regulations. Description of tests performed to search for material weaknesses. Restriction on the use of the report

Explanation Choice "4" is correct. Any report issued on significant deficiencies noted during an audit should (1) indicate that the purpose of the audit was to report on the financial statements and not to provide assurance on internal control, (2) include the definition of significant deficiencies, and (3) include a restriction on the use of the report. Choice "1" is incorrect. There is no paragraph describing management's evaluation in a letter issued on significant deficiencies. Choice "2" is incorrect. There is no required statement of compliance with applicable laws and regulations in a letter issued on significant deficiencies. Choice "3" is incorrect. Because the auditor does not search for material weaknesses, a description of tests performed in this regard would not be included in a written report.

The auditor may choose to apply substantive tests to balance sheet accounts at an interim period: For accounts that are reasonably unpredictable with respect to amount, relative significance, and composition. When there are rapidly changing business conditions. In an effort to reduce detection risk. When inherent risk and control risk are low.

Explanation Choice "4" is correct. Applying substantive audit tests at interim increases detection risk, but this may be acceptable when the risk of material misstatement (comprised of inherent risk and control risk) is low. Choice "1" is incorrect. Interim testing is more likely to be performed for accounts that are reasonably predictable (not unpredictable) with respect to amount, relative significance, and composition. Choice "2" is incorrect. Rapidly changing business conditions make year-end account balances less predictable, making it less likely that the auditor would choose to apply substantive tests at interim. Choice "3" is incorrect. Applying substantive audit tests at interim increases detection risk.

Which of the following is true when comparing a compilation engagement and a review engagement for a nonissuer? Both compilations and reviews provide some form of assurance on financial statements. In a compilation, only limited assurance is provided, whereas in a review, positive assurance is provided. Negative assurance is expressed in a compilation, but not in a review. Both compilations and reviews require establishment of an understanding with the client.

Explanation Choice "4" is correct. Both compilations and reviews require the establishment of an understanding with the client. Choice "1" is incorrect. No assurance at all is provided in a compilation. Choice "2" is incorrect. No assurance is provided in a compilation, whereas limited (or negative) assurance is provided in a review. Choice "3" is incorrect. Negative assurance is expressed in a review, but not in a compilation.

Davis, CPA compiled financial statements for XYZ Co. as of and for the year ended December 31, Year 7 that omitted substantially all disclosures required by generally accepted accounting principles (GAAP). Davis was asked to compile the financial statements again for the year ended December 31, Year 8 and to include all necessary disclosures. XYZ Co. plans to prepare the Year 8 financial statements in comparative form. In this situation, Davis may: Issue two separate reports (separate compilation reports for Year 7 and Year 8) so that a clear indication may be provided that one year contains substantially all disclosures and the other does not. Issue a standard compilation report on the current year (Year 8) and add a paragraph to the prior year (Year 7) compilation report indicating that the financial statements omitted substantially all disclosures Issue an adverse opinion or a qualified (except for) opinion for lack of adequate disclosure. Not issue a report on comparative financial statements when statements for some, but not all, of the periods presented omit substantially all of the disclosures required by GAAP.

Explanation Choice "4" is correct. Compiled financial statements that omit substantially all GAAP disclosures are not comparable to financial statements including such disclosures. The accountant should not issue a report in this situation. Choices "1", "2" or "3" are incorrect. Professional standards indicate that no report should be issued in such situations.

A report on a nonissuer's internal control based on an audit of internal control should include a statement indicating that: I. Because there are inherent limitations in internal control, misstatements may occur and not be detected. II. Projections of the evaluation of internal control to future periods are subject to the risk that the internal control may become inadequate. III. The report is intended solely for the information and use of the audit committee, management, and other specified parties. II and III, only. I, II, and III. I and III only. I and II only.

Explanation Choice "4" is correct. Each of the first two statements would be included in the inherent limitations paragraph of the report. The third statement, indicating restricted use, would not be included as there is no restriction on the use of this report. (A report on internal control matters noted during an audit would, however, include a restricted use paragraph.) Choices "3", "2", and "1" are incorrect, based on the above explanation.

A limitation on the scope of an audit sufficient to preclude an unmodified opinion will usually result when the client: Omits the statement of cash flows. Refuses to disclose in the notes to the financial statements a significant related party transaction. Asks the auditor to report on the balance sheet and not the other basic financial statements. Does not make the minutes of the Board of Directors meetings available to the auditor.

Explanation Choice "4" is correct. Failure to make the minutes available is a scope limitation sufficient to preclude an unmodified opinion. Choices "1" and "2" are incorrect. Omission of the statement of cash flows and refusal to disclose a significant related party transaction are examples of GAAP violations, but they do not constitute a limitation on the scope of the audit. Choice "3" is incorrect. Reporting on only one financial statement and not the others simply involves a limited reporting objective. The auditor should obtain an understanding of the purpose for which the financial statement is prepared, the intended users, and the steps taken by management to determine that the applicable financial reporting framework is acceptable in the circumstances. As long as these steps are taken by the auditor, auditing a single financial statement would not constitute a limitation on the scope of the audit.

Which of the following procedures are required for a compilation engagement? I. Read the financial statements. II. Obtain an understanding of the client's business and the accounting principles used. III. Assess fraud risk. II and III, only. I and III, only. I, II, and III. I and II, only.

Explanation Choice "4" is correct. In a compilation engagement, the accountant is required to read the financial statements and to have a general understanding of the client's business and of the accounting principles and practices used by the client. There is no requirement that the accountant specifically assess fraud, although a discovery of fraud cannot be ignored. Choices "3", "2", and "1" are incorrect, based on the above explanation

With respect to an auditor's consideration of fraud risk, which of the following is not required? An assumption that that there is a risk of management override of controls. Incorporation of an element of unpredictability into the audit. A discussion among engagement personnel regarding fraud. Implementation of controls to detect fraud.

Explanation Choice "4" is correct. It is management's responsibility (not the auditor's) to design and implement programs and controls to prevent, deter, and detect fraud. Choice "1" is incorrect. There is a presumption in every audit that risk of management override of controls exists. Choice "2" is incorrect. The auditor should incorporate an element of unpredictability into every audit. Choice "3" is incorrect. During planning, engagement personnel are required to discuss the potential for material misstatement due to fraud.

All of the following are effective ways to prevent and/or detect lapping, except for: Requiring that customers send their payments directly to a lock box. Comparing the dollar amounts and dates on the bank deposit slips with customer remittance credits entered into the accounts receivable ledger. Independently comparing the recorded cash receipts with funds actually deposited in the bank. Preparing a bank transfer schedule.

Explanation Choice "4" is correct. Lapping is withholding and not recording the current receipt of cash or checks, which is covered by using a subsequent receipt for the previous unrecorded amount. Preparing a bank transfer schedule is an effective means to detect kiting, not lapping. Choices "2", "1", and "3" are incorrect as each represents an effective way to prevent and/or detect lapping.

A report issued on significant deficiencies in internal control noted during a financial statement audit of a nonissuer should contain all of the following except: A restriction on the use of the report. The definition of material weaknesses. An indication that the purpose of the audit was to report on the financial statements. A statement of compliance with laws and regulations.

Explanation Choice "4" is correct. No statement of compliance with laws and regulations is required in the report. Choices "2", "1", and "3" are incorrect. The definition of material weaknesses, a restriction on the use of the report, and an indication that the purpose of the audit was to report on the financial statements all should be part of the report.

Which of the following is true about the assurance provided by special reports? All special reports result in positive assurance. All special reports result in negative assurance. A list of procedures and findings (but no assurance) is provided in some special reports. A positive opinion may be rendered in some types of special reports.

Explanation Choice "4" is correct. Positive assurance is provided in OCBOA reports, reports on specified elements, accounts, or items, and reports on special-purpose financial presentations to comply with contractual agreements or regulatory provisions. Where applicable, the emphasis-of-matter paragraph in the report indicates that the financial statements (elements, accounts, items) were prepared in accordance with the applicable special purpose framework (positive assurance). Choice "1" is incorrect. Negative assurance is provided in reports on compliance with aspects of contractual agreements or regulatory requirements related to audited financial statements. Choice "2" is incorrect. Positive assurance is provided in OCBOA reports, reports on specified elements, accounts, or items, and reports on special-purpose financial presentations to comply with contractual agreements or regulatory provisions. Choice "3" is incorrect. Procedures and findings are listed in reports related to agreed-upon procedures engagements, not in special reports.

In an audit of a nonissuer's financial statements, projected misstatement is: The likely amount of misstatement in the subsequent-period's financial statements if a control is not properly implemented. The only amount that the auditor considers in evaluating materiality and fairness of the financial statements. An auditor's best estimate, before performing audit procedures, of misstatements that the auditor expects to find during the audit. An auditor's best estimate of misstatements in a population extrapolated from misstatements identified in an audit sample.

Explanation Choice "4" is correct. Projected misstatement is an auditor's best estimate of misstatements in a population extrapolated from misstatements identified in an audit sample. Choice "1" is incorrect. Projected misstatement is an auditor's best estimate of misstatements in a population extrapolated from misstatements identified in an audit sample for the period under audit. It does not relate to the estimated amount of misstatement in the subsequent-period's financial statements if a control is not properly implemented. Choice "2" is incorrect. In addition to projected misstatements, an auditor takes into consideration factual and judgmental misstatements when evaluating materiality and fairness of the financial statements. (Note: Be careful when selecting answers that contain absolute terms such as all, always, only, etc.) Choice "3" is incorrect. Expected misstatement is the auditor's best estimate, before performing audit procedures, of misstatements that the auditor expects to find during the audit.

Janus Company, a client of Peterman, CPA, has recently automated its accounting system. While Peterman has been the auditor of Janus for several years, this will be the first audit encompassing the new system. Which of the following is most likely a result of this change? I. Peterman will need to take courses to develop an appropriate level of IT skill. II. Peterman will need to revise his audit objectives from prior years to reflect the new situation. III. Peterman will need to revise his audit program from prior years to reflect the new situation. Both I and II. I, II, and III. Both II and III. III only.

Explanation Choice "4" is correct. The audit program will likely need to be revised to reflect the risks and capitalize on the strengths inherent in an automated system. For example, there will likely be a greater risk of unauthorized access, while there may also be greater opportunities for data analysis and review. Audit objectives are the same in a computerized environment as they are in a manual environment. If specialized IT skills are needed, the auditor is less likely to take IT courses than he or she is to seek the assistance of an IT professional. Choices "3", "1", and "2" are incorrect, based on the above explanation.

In which situations should disagreements with management and significant audit adjustments be communicated to those charged with governance? Disagreements with Management Significant Audit Adjustments Communicate only if the issue has not been satisfactorily resolved. Communicate regardless of whether the adjustment has been booked Communicate even if the issue has been satisfactorily resolved Communicate only if management declines to book the adjustment Communicate only if the issue has not been satisfactorily resolved Communicate only if management declines to book the adjustment Communicate even if the issue has been satisfactorily resolved Communicate regardless of whether the adjustment has been booked

Explanation Choice "4" is correct. The auditor should discuss with those charged with governance any significant disagreements with management, whether or not satisfactorily resolved. The auditor should also inform those charged with governance about significant adjustments arising from the audit, regardless of whether such adjustments were recorded by the client. Choices "3", "2", and "1" are incorrect based on the above explanation.

When an auditor believes there is substantial doubt about the ability of an entity to continue as a going concern, all of the following should be included in the audit documentation, except: The conditions that gave rise to the substantial doubt. The auditor's conclusion about whether substantial doubt remains or is alleviated. The effect of the auditor's conclusion on the auditor's report. Mitigating factors considered insignificant.

Explanation Choice "4" is correct. The auditor should include in the audit documentation any mitigating factors that the auditor considers significant (not insignificant). Choices "1", "2", and "3" are incorrect. When an auditor believes there is substantial doubt about the ability of an entity to continue as a going concern, the conditions that gave rise to the substantial doubt, the auditor's conclusion about whether substantial doubt remains or is alleviated, and the effect of the auditor's conclusion on the auditor's report should all be documented.

Which of the following auditing procedures most likely would assist an auditor in identifying related party transactions? Performing analytical procedures for indications of possible financial difficulties. Reviewing the accounting records for usual or recurring transactions or balances. Inspecting correspondence with lawyers for evidence of unreported contingent liabilities. Reading the minutes of the Board of Directors meetings to see if any material transactions were authorized

Explanation Choice "4" is correct. The auditor should review the minutes of meetings of the Board of Directors, since material related party transactions may be authorized or discussed during those meetings. Choice "1" is incorrect. Analytical procedures might indicate adverse financial trends, but they would not be specific enough to assist the auditor in identifying specific related party transactions. Choice "2" is incorrect. The accounting records should be reviewed for unusual or nonrecurring transactions or balances. Choice "3" is incorrect. Inspecting correspondence with lawyers might provide evidence regarding unreported contingent liabilities, but this would not aid the auditor in identifying specific related party transactions.

When a client makes extensive use of information technology, the auditor should consider the effect this may have on internal control. Which of the following is least likely to be affected? The audit procedures used to evaluate controls. The assessed level of control risk. The five components of internal control. The audit objectives with respect to evaluating internal control.

Explanation Choice "4" is correct. The client's extensive use of information technology generally would not affect the auditor's objectives, although it might affect how those objectives are achieved. Choice "1" is incorrect. An entity's use of information technology will affect the appropriate audit procedures to apply. For example, the extent and complexity of computer operations may require the use of computer-assisted audit techniques. Choice "2" is incorrect. An entity's use of information technology may create additional internal control risks, such as the risk of unauthorized access to data. Choice "3" is incorrect. An entity's use of information technology may affect any of the five components of internal control.

Proper authorization of write-offs of uncollectible accounts should be approved in which of the following departments? Accounts payable. Credit. Accounts receivable. Treasurer.

Explanation Choice "4" is correct. The treasurer does not perform duties that are incompatible with authorizing write-offs since he or she is usually not involved with sales transactions or recordkeeping. Choice "1" is incorrect. The accounts payable department is typically involved in the expenditure cycle, not the revenue cycle. Choice "2" is incorrect. Granting credit and authorizing write-offs represents an improper segregation of duties since non-existent customers could have credit authorized and then have their accounts written off. Choice "3" is incorrect. Recording accounts receivable and authorizing write-offs would constitute an improper segregation of duties.

An auditor makes use of a specialist who has a contractual relationship with the client. Which of the following is true? The lack of independence must be disclosed in the auditor's report, but will not preclude the issuance of an unqualified opinion. This is a violation of generally accepted auditing standards. The lack of independence must be disclosed in the auditor's report, and the auditor should issue a qualified opinion or a disclaimer of opinion, depending on materiality. This is acceptable provided that the auditor evaluates the effect of the relationship on the specialist's objectivity.

Explanation Choice "4" is correct. Use of a specialist who is related to the client is acceptable, but the auditor must consider the risk that the specialist's objectivity might be impaired, in which case the auditor may need to perform additional procedures. Choice "2" is incorrect. Use of a specialist who is related to the client is acceptable and is not a violation of generally accepted auditing standards. Choices "3" and "1" are incorrect. The fact that the specialist is related to the client need not be disclosed in the audit report, and it will not preclude the issuance of an unqualified opinion.

Based on new information gained during an audit of a nonissuer, an auditor determines that it is necessary to modify materiality for the financial statements as a whole. In this circumstance, which of the following statements is accurate? The revision of materiality at the financial statement levels will not affect the planned nature and timing of audit procedures, only the extent of those procedures. The auditor should consider disclaiming an opinion due to a scope limitation. The auditor is required to reperform audit procedures already completed on the audit using the revised materiality. Materiality levels for particular classes of transactions, account balances, or disclosures might also need to be revised.

Explanation Choice "4" is correct. When materiality for the financial statements as a whole are revised, the materiality levels for particular classes of transactions, account balances, or disclosures may also need to be revised. Choice "1" is incorrect. The revision of materiality at the financial statement levels may also affect the planned nature and timing of audit procedures. (Note: Be careful when selecting answers that contain absolute terms such as all, always, only, etc.) Choice "2" is incorrect. A revision of materiality is not considered a limitation on the scope of the engagement. Choice "3" is incorrect. An auditor is not required to reperform audit procedures already completed on the audit using the revised materiality. However, a revised materiality may affect the planned nature, extent and timing of audit procedures. For example, the sample size may increase as a result of a revised materiality. The auditor may use the items already selected and then just select additional items to meet the increased sample size

Which of the following is least likely to explain an increase in inventory turnover from Year 1 to Year 2? Purchases received on December 31 of Year 2 were not recorded until January 1 of Year 3. Returned goods received during Year 2 were not recorded. Direct labor charges were inadvertently applied twice to some inventory items during Year 2. Items shipped on consignment during Year 2 were mistakenly recorded as sales.

Explanation Remember that inventory turnover is computed as cost of goods sold divided by average inventory. Also note that an error in sales typically results in a corresponding error in cost of goods sold (i.e., if sales are overstated, cost of goods sold is likely to be overstated as well.). Choice "3" is correct. An overstatement of inventory in Year 2 would result in a decline in the inventory turnover ratio, not an increase. Choice "1" is incorrect. An understatement of purchases results in an understatement of inventory, which in turn results in an increase in inventory turnover. Choice "2" is incorrect. Failure to record returned goods results in an understatement of inventory and an overstatement of sales, both of which would result in an increase in inventory turnover. Choice "4" is incorrect. The error described results in an understatement of inventory and an overstatement of sales, both of which would result in an increase in inventory turnover.


संबंधित स्टडी सेट्स

Chem 1320 Final Exam (Multiple Choice)

View Set

List of Mobile Phone Operating Systems

View Set

Module 02 (part 1): Digital Devices

View Set

Module 10 - Drugs and Consciousness - Test

View Set

Greek Civ: Influence of Athenian Democracy on the US

View Set

Integumentary System (Lecture 1/2)

View Set

Chapter 21: Title, Risk, and Insurable Interest

View Set